LSAT and Law School Admissions Forum

Get expert LSAT preparation and law school admissions advice from PowerScore Test Preparation.

 Administrator
PowerScore Staff
  • PowerScore Staff
  • Posts: 8916
  • Joined: Feb 02, 2011
|
#61023
Please post your questions below!
 Bruin96
  • Posts: 33
  • Joined: Sep 04, 2019
|
#72117
Hello,

I initially got this question wrong but I still do not understand why answer choice C would be incorrect?

If K & L are placed on one day, then M & N block would be placed on another day. The set up I had was

Group 1:
J
M
N

Group 2:
K
L
O

Thus, answer choice C is valid because K & L cannot go together.
 Claire Horan
PowerScore Staff
  • PowerScore Staff
  • Posts: 408
  • Joined: Apr 18, 2016
|
#72135
Hi Bruin96,

The question asks for a pair of speakers who cannot speak together on the same day. So, all of the incorrect answer pairs will have at least one solution in which they can speak on the same day. You've shown one combination where K and L are on the same day and the rules are violated, and from that you've concluded that they can NEVER be placed on the same day. But there are may be other combinations with K and L on the same day that follow the rules.

Here's one:

Thursday:
1pm J
2pm K
3pm L

Friday:
1pm N
2pm M
3pm O

You should verify for yourself that all the solution above follows all the rules. At that point, you can eliminate answer choice C because you have found a counterexample where K and L are on the same day.

I hope this explanation helps! Please post again if you still have questions!
 Bruin96
  • Posts: 33
  • Joined: Sep 04, 2019
|
#72166
Thank you, Claire! I just revisited this problem with fresh eyes and got it correct! I am starting to realize that I am limiting my setups and missing a point or two on games because I do not see other possibilities.

Get the most out of your LSAT Prep Plus subscription.

Analyze and track your performance with our Testing and Analytics Package.